Guyton Test #1 Flashcards

1
Q

What is the Stroke volume in milliliters?

A

100 mL

How well did you know this?
1
Not at all
2
3
4
5
Perfectly
2
Q

What is the cardiac output of this patient?

A

8200 mL/min

How well did you know this?
1
Not at all
2
3
4
5
Perfectly
3
Q

What is the extent of diastole in the ventricular pressure-volume relationship

A

From point D and Point B

How well did you know this?
1
Not at all
2
3
4
5
Perfectly
4
Q

What is correct about isovolumetric contraction?

A

Extends from B to C in the ventricular pressure-volume curve

How well did you know this?
1
Not at all
2
3
4
5
Perfectly
5
Q

A 47-year-old man has an ejection fraction of 0.32 and an end- diastolic volume of 160 ml. What is (approximately) the value of end-systolic volume?

A

51 ml

How well did you know this?
1
Not at all
2
3
4
5
Perfectly
6
Q

In a resting adult, the typical ventricular ejection fraction has what value?

A) 20%
B) 30%
C) 40%
D) 60%
E) 80%

A

D) 60%

How well did you know this?
1
Not at all
2
3
4
5
Perfectly
7
Q

What happens at the end of ventricular isovolumic relaxation?

A) The A-V valves close
B) The aortic valve opens
C) The aortic valve closes
D) The mitral valve opens
E) The pulmonary valve closes

A

D) The mitral valve opens

How well did you know this?
1
Not at all
2
3
4
5
Perfectly
8
Q

Which event is associated with the first heart sound?

A) Closing of the aortic valve
B) Inrushing of blood into the ventricles during diastole
C) Beginning of diastole
D) Opening of the A-V valves
E) Closing of the A-V valves

A

E) Closing of the A-V valves

How well did you know this?
1
Not at all
2
3
4
5
Perfectly
9
Q

Which condition will result in a dilated, flaccid heart?

A) Excess calcium ions in the blood
B) Excess potassium ions in the blood
C) Excess sodium ions in the blood
D) Increased sympathetic stimulation
E) Increased norepinephrine concentration in the blood

A

B) Excess potassium ions in the blood

How well did you know this?
1
Not at all
2
3
4
5
Perfectly
10
Q

A 25-year-old well-conditioned athlete weighs 80 kg (176 lb). During maximal sympathetic stimulation, what is the plateau level of his cardiac output function curve?

A) 3 l/min
B) 5 l/min
C) 10 l/min
D) 13 l/min
E) 25 l/min

A

E) 25 l/min

How well did you know this?
1
Not at all
2
3
4
5
Perfectly
11
Q

A 25-year-old well-conditioned athlete weighs 80 kg (176 lb).
During maximal sympathetic stimulation, what is the plateau level of his cardiac output function curve?

A) 3 l/min
B) 5 l/min
C) 10 l/min
D) 13 l/min
E) 25 l/min

A

E) 25 l/min

How well did you know this?
1
Not at all
2
3
4
5
Perfectly
12
Q

Which phase of the cardiac cycle follows immediately after the beginning of the QRS wave?

A) Isovolumic relaxation
B) Ventricular ejection
C) Atrial systole
D) Diastasis
E) Isovolumic contraction

A

E) Isovolumic contraction

How well did you know this?
1
Not at all
2
3
4
5
Perfectly
13
Q

Sympathetic stimulation of the heart does which of the following?

A) Releases acetylcholine at the sympathetic endings
B) Decreases sinus nodal discharge rate
C) Decreases excitability of the heart
D) Releases norepinephrine at the sympathetic endings
E) Decreases cardiac contractility

A

D) Releases norepinephrine at the sympathetic endings

How well did you know this?
1
Not at all
2
3
4
5
Perfectly
14
Q

Sympathetic stimulation of the heart normally causes which condition?

A) Acetylcholine release at the sympathetic endings
B) Decreased heart rate
C) Decreased rate of conduction of the cardiac impulse
D) Decreased force of contraction of the atria
E) Increased force of contraction of the ventricles

A

E) Increased force of contraction of the ventricles

How well did you know this?
1
Not at all
2
3
4
5
Perfectly
15
Q

What is correct about the sinus node?

A) Delays the cardiac conduction if sympathetic activity
increases
B) Acts as a pacemaker because the membrane constantly leaks
Na+ from extracellular fluid
C) The constant leak of K+ makes resting potential in the sinus
node gradually rise
D) Feedback from the Purkinje fibers defines the sinus node
discharge
E) The resting membrane potential of the sinus node is +55 to
+60mV

A

B) Acts as a pacemaker because the membrane constantly leaks
Na+ from extracellular fluid

How well did you know this?
1
Not at all
2
3
4
5
Perfectly
16
Q

Listed below are the hydrostatic and oncotic pressures within a microcirculatory bed.

Plasma colloid osmotic pressure = 25 mm Hg Capillary hydrostatic pressure = 25 mm Hg
Venous hydrostatic pressure = 5 mm Hg Arterial pressure = 80 mm Hg
Interstitial fluid hydrostatic pressure = −5 mm Hg Interstitial colloid osmotic pressure = 10 mm Hg Capillary filtration rate = 150 ml/min

What is the capillary filtration coefficient (in ml/min/mm Hg) for this capillary wall?

A) 5
B) 10
C) 15
D) 20
E) 25

A

B) 10

How well did you know this?
1
Not at all
2
3
4
5
Perfectly
17
Q

A healthy 60-year-old woman with a 10-year history of hypertension stands up from a supine position. Which set of cardiovascular changes is most likely to occur in response to standing up from a supine position?

A

A) Increase, increase, increase

How well did you know this?
1
Not at all
2
3
4
5
Perfectly
18
Q

In an experimental study, administration of a drug decreases the diameter of arterioles in the muscle bed of an animal subject.

Which set of physiological changes would be expected to occur in response to the decrease in diameter?

A

E.) decrease, decrease, decrease

How well did you know this?
1
Not at all
2
3
4
5
Perfectly
19
Q

A 60-year-old woman has experienced dizziness for the past 6 months when getting out of bed in the morning and standing up.

Her mean arterial pressure is 103/90 while lying down and 95/60 when standing.

Which set of physiological changes would be expected in response to moving from a supine to an upright position.

A

G) decrease, increase, increase

How well did you know this?
1
Not at all
2
3
4
5
Perfectly
20
Q

A 35-year-old woman visits her family practitioner for an examination.

She has a blood pressure of 160/75 mm Hg and a heart rate of 74 beats/min.

Further tests by a cardiologist reveal that the patient has moderate aortic regurgitation. Which set of changes would be expected in this patient?

A

A) increase, increase, increase

How well did you know this?
1
Not at all
2
3
4
5
Perfectly
21
Q

A healthy 27-year-old female medical student runs a 5K race.

Which set of physiological changes is most likely to occur in this woman’s skeletal muscles during the race?

A

F) decrease, decrease, increase

How well did you know this?
1
Not at all
2
3
4
5
Perfectly
22
Q

Cognitive stimuli such as reading, problem solving, and talking all result in significant increases in cerebral blood flow.

Which set of changes in cerebral tissue concentrations is the most likely explanation for the increase in cerebral blood flow?

A

B) increase, decrease, increase

How well did you know this?
1
Not at all
2
3
4
5
Perfectly
23
Q

Histamine is infused into the brachial artery. Which set of microcirculatory changes would be expected in the infused arm?

A

A) increase, increase, increase

How well did you know this?
1
Not at all
2
3
4
5
Perfectly
24
Q

An increase in shear stress in a blood vessel results in which change?

A) Decreased endothelin production
B) Decreased cyclic guanosine monophosphate production
C) Increased nitric oxide release
D) Increased renin production
E) Decreased prostacyclin production

A

C) Increased nitric oxide release

How well did you know this?
1
Not at all
2
3
4
5
Perfectly
25
Q

A 65-year-old man with a 10-year history of essential hypertension is being treated with an angiotensin-converting enzyme (ACE) inhibitor. Which set of changes would be expected to occur in response to the ACE inhibitor drug therapy?

A

C) increase, decrease, decrease

How well did you know this?
1
Not at all
2
3
4
5
Perfectly
26
Q

The diameter of a precapillary arteriole is decreased in a muscle vascular bed. An increase in which of the following would be expected?

A) Capillary filtration rate
B) Vascular conductance
C) Capillary blood flow
D) Capillary hydrostatic pressure
E) Arteriolar resistance

A

E) Arteriolar resistance

How well did you know this?
1
Not at all
2
3
4
5
Perfectly
27
Q

A 55-year-old man with a history of normal health visits his physician for a checkup. The physical examination reveals that his blood pressure is 170/98 mm Hg. Further tests indicate that he has renovascular hypertension as a result of stenosis in the left kidney. Which set of findings would be expected in this man with renovascular hypertension?

A

A) increase, increase, increase

How well did you know this?
1
Not at all
2
3
4
5
Perfectly
28
Q

Under control conditions, flow through a blood vessel is 100 ml/min with a pressure gradient of 50 mm Hg.

What would be the approximate flow through the vessel after increasing the vessel diameter by 100%, assuming that the pressure gradient is maintained at 50 mm Hg?

A) 200 ml/min
B) 400 ml/min
C) 800 ml/min
D) 1600 ml/min
E) 700 ml/min

A

D) 1600 ml/min

How well did you know this?
1
Not at all
2
3
4
5
Perfectly
29
Q

While participating in a cardiovascular physiology laboratory, a medical student isolates an animal’s carotid artery proximal to the carotid bifurcation and partially constricts the artery with a tie around the vessel.

Which set of changes would be expected to occur in response to constriction of the carotid artery?

A

D) increase, decrease, increase

How well did you know this?
1
Not at all
2
3
4
5
Perfectly
30
Q

A 35-year-old woman visits her family practice physician for an examination.

She has a mean arterial blood pressure of 105 mm Hg and a heart rate of 74 beats/min.

Further tests by a cardiologist reveal that the patient has moderate aortic valve stenosis. Which set of changes would be expected in this patient?

A

E) decrease, decrease, decrease

How well did you know this?
1
Not at all
2
3
4
5
Perfectly
31
Q

A 60-year-old man visits his family practitioner for an annual examination. He has a mean blood pressure of 130 mm Hg and a heart rate of 78 beats/min. His plasma cholesterol level is in the upper 25th percentile, and he is diagnosed as having atherosclerosis. Which set of changes would be expected in this patient?

A

B) increase, decrease, increase,

How well did you know this?
1
Not at all
2
3
4
5
Perfectly
32
Q

While participating in a cardiovascular physiology laboratory, a medical student isolates the carotid artery of an animal and partially constricts the artery with a tie around the vessel. Which set of changes would be expected to occur in response to constriction of the carotid artery?

A

B) increase, decrease, increase

How well did you know this?
1
Not at all
2
3
4
5
Perfectly
33
Q

Which one of the following would tend to increase capillary filtration rate?

A) Decreased capillary hydrostatic pressure
B) Decreased plasma colloid osmotic pressure
C) Decreased interstitial colloid osmotic pressure
D) Decreased capillary water permeability
E) Increased arteriolar resistance

A

B) Decreased plasma colloid osmotic pressure

How well did you know this?
1
Not at all
2
3
4
5
Perfectly
34
Q

A decrease in which of the following would tend to increase lymph flow?

A) Hydraulic conductivity of the capillary wall
B) Plasma colloid osmotic pressure
C) Capillary hydrostatic pressure
D) Vascular conductance
E) B and D

A

B) Plasma colloid osmotic pressure

How well did you know this?
1
Not at all
2
3
4
5
Perfectly
35
Q

Under control conditions, flow through a blood vessel is 100 ml/min under a pressure gradient of 50 mm Hg. What would be the approximate flow through the vessel after increasing the vessel diameter to four times normal, assuming that the pressure gradient was maintained at 50 mm Hg?

A) 200 ml/min
B)400 ml/min
C) 800 ml/min
D) 1600 ml/min
E) 700 ml/min

A

E) 1600 ml/min

How well did you know this?
1
Not at all
2
3
4
5
Perfectly
36
Q

An increase in which of the following would be expected to decrease blood flow in a vessel?

A) Pressure gradient across the vessel
B) Radius of the vessel
C) Plasma colloid osmotic pressure
D) Viscosity of the blood
E)Plasma sodium concentration

A

D) Viscosity of the blood

How well did you know this?
1
Not at all
2
3
4
5
Perfectly
37
Q

Assuming that vessels A to D are the same length, which one has
the greatest flow?

A

D) 10, 6, 1

How well did you know this?
1
Not at all
2
3
4
5
Perfectly
38
Q

A healthy 28-year-old woman stands up from a supine position. Moving from a supine to a standing position results in a transient decrease in arterial pressure that is detected by arterial baroreceptors located in the aortic arch and carotid sinuses. Which set of cardiovascular changes is most likely to occur in response to activation of the baroreceptors?

A

A) increase, increase, increase

How well did you know this?
1
Not at all
2
3
4
5
Perfectly
39
Q

Under normal physiological conditions, blood flow to the skeletal muscles is determined mainly by which of the following?

A) Sympathetic nerves
B) Angiotensin II
C) Vasopressin
D) Local metabolic factors
E) Capillary osmotic pressure

A

D) Local metabolic factors

How well did you know this?
1
Not at all
2
3
4
5
Perfectly
40
Q

A healthy 22-year-old female medical student has an exercise stress test at a local health club. An increase in which of the following is most likely to occur in this woman’s skeletal muscles during exercise?

A) Vascular conductance
B) Blood flow
C) Carbon dioxide concentration D) Arteriolar diameter
E) All the above

A

E) All the above

How well did you know this?
1
Not at all
2
3
4
5
Perfectly
41
Q

Which of the following segments of the circulatory system has the lowest velocity of blood flow?

A) Aorta
B) Arteries
C) Capillaries
D) Veins

A

C) Capillaries

How well did you know this?
1
Not at all
2
3
4
5
Perfectly
42
Q

Listed below are the hydrostatic and oncotic pressures within a microcirculatory bed.

Plasma colloid osmotic pressure = 25 mm Hg Capillary hydrostatic pressure = 25 mm Hg Venous hydrostatic pressure = 5 mm Hg Arterial pressure = 80 mm Hg
Interstitial hydrostatic pressure = −5 mm Hg Interstitial colloid osmotic pressure = 5 mm Hg Filtration coefficient = 15 ml/min/mm Hg

What is the filtration rate (ml/min) of the capillary wall?
A) 100
B) 150
C) 200
D) 250
E) 300

A

B) 150

How well did you know this?
1
Not at all
2
3
4
5
Perfectly
43
Q

Which blood vessel has the highest vascular resistance?

A

A) BF = 1000; PG = 100

How well did you know this?
1
Not at all
2
3
4
5
Perfectly
44
Q

Which of the following vessels has the greatest total cross- sectional area in the circulatory system?

A) Aorta
B) Small arteries
C) Capillaries
D) Venules
E) Vena cava

A

C) Capillaries

How well did you know this?
1
Not at all
2
3
4
5
Perfectly
45
Q

A balloon catheter is advanced from the superior vena cava into the heart and inflated to increase atrial pressure by 5 mm Hg.
Which of the following would be expected to occur in response to the elevated atrial pressure?

A) Decreased atrial natriuretic peptide
B) Increased angiotensin II
C) Increased aldosterone
D) Decreased renal sympathetic nerve activity

A

D) Decreased renal sympathetic nerve activity

How well did you know this?
1
Not at all
2
3
4
5
Perfectly
46
Q

An increase in atrial pressure results in which of the following?

A) Increased plasma atrial natriuretic peptide
B) Increase in plasma angiotensin II concentration
C) Decrease in plasma aldosterone concentration
D) Decrease in sodium excretion
E) A and C

A

E) A and C

How well did you know this?
1
Not at all
2
3
4
5
Perfectly
47
Q

Autoregulation of tissue blood flow in response to an increase in arterial pressure occurs as a result of which of the following?

A) Decrease in vascular resistance
B) Initial decrease in vascular wall tension
C) Excess delivery of nutrients such as oxygen to the tissues
D) Decrease in tissue metabolism

A

C) Excess delivery of nutrients such as oxygen to the tissues

How well did you know this?
1
Not at all
2
3
4
5
Perfectly
48
Q

Which component of the circulatory system contains the largest percentage of the total blood volume?
A) Arteries
B) Capillaries
C) Veins
D) Pulmonary circulation E) Heart

A

C) Veins

How well did you know this?
1
Not at all
2
3
4
5
Perfectly
49
Q

An increase in which of the following tends to decrease capillary filtration rate?

A) Capillary hydrostatic pressure
B) Plasma colloid osmotic pressure
C) Interstitial colloid osmotic pressure
D) Venous hydrostatic pressure
E) Arteriolar diameter

A

B) Plasma colloid osmotic pressure

How well did you know this?
1
Not at all
2
3
4
5
Perfectly
50
Q

Which set of changes would be expected to occur 2 weeks after a 50% reduction in renal artery pressure?

A

B) increase, increase, decrease

How well did you know this?
1
Not at all
2
3
4
5
Perfectly
51
Q

Which of the following would tend to increase lymph flow?

A) Increase capillary hydrostatic pressure
B) Increased plasma colloid osmotic pressure
C) Increased interstitial volume
D) Decreased arteriolar diameter
E) A and C

A

E) A and C

How well did you know this?
1
Not at all
2
3
4
5
Perfectly
52
Q

An increase in the production of which of the following would
most likely result in chronic hypertension?

A) Aldosterone
B) Prostacyclin
C) Angiotensin II
D) Nitric oxide
E) A and C

A

E) A and C

How well did you know this?
1
Not at all
2
3
4
5
Perfectly
53
Q

A decrease in which of the following would be expected to occur in a person 2 weeks after an increase in sodium intake?

A) Angiotensin II
B) Sodium Excretion
C) Aldosterone
D) Atrial natriuretic peptide E) A and C

A

E) A and C

How well did you know this?
1
Not at all
2
3
4
5
Perfectly
54
Q

Which of the following would be expected to occur during a Cushing reaction caused by brain ischemia?

A) Increase in parasympathetic activity
B) Decrease in arterial pressure
C) Decrease in heart rate
D) Increase in sympathetic activity

A

D) Increase in sympathetic activity

How well did you know this?
1
Not at all
2
3
4
5
Perfectly
55
Q

A 65-year-old man has congestive heart failure. He has a cardiac output of 4 l/min, arterial pressure of 115/85 mm Hg, and heart rate of 90 beats/min.

Further tests by a cardiologist reveal that the patient has a right atrial pressure of 10 mm Hg. An increase in which of the following would be expected in this patient?

A) Plasma colloid osmotic pressure
B) Interstitial colloid osmotic pressure
C) Arterial pressure
D) Cardiac output
E) Vena cava hydrostatic pressure

A

E) Vena cava hydrostatic pressure

How well did you know this?
1
Not at all
2
3
4
5
Perfectly
56
Q

Which part of the circulation has the highest compliance?

A) Capillaries
B) Large arteries
C) Veins
D) Aorta
E) Small arteries

A

C) Veins

How well did you know this?
1
Not at all
2
3
4
5
Perfectly
57
Q

An increase in which of the following tends to increase pulse pressure?

A) Systolic pressure
B) Capillary hydrostatic pressure
C) Arterial compliance
D) Stroke volume
E) A and D

A

E) A and D

How well did you know this?
1
Not at all
2
3
4
5
Perfectly
58
Q

Which of the following would be expected to occur during a Cushing reaction caused by brain ischemia?

A) Increase in parasympathetic activity
B) Decrease in arterial pressure
C) Decrease in heart rate
D) Increase in sympathetic activity

A

C) Decrease in heart rate

How well did you know this?
1
Not at all
2
3
4
5
Perfectly
59
Q

A 65-year-old man has congestive heart failure. He has a cardiac output of 4 l/min, arterial pressure of 115/85 mm Hg, and heart rate of 90 beats/min. Further tests by a cardiologist reveal that the patient has a right atrial pressure of 10 mm Hg. An increase in which of the following would be expected in this patient?

A) Plasma colloid osmotic pressure
B) Interstitial colloid osmotic pressure
C) Arterial pressure
D) Cardiac output
E) Vena cava hydrostatic pressure

A

E) Vena cava hydrostatic pressure

How well did you know this?
1
Not at all
2
3
4
5
Perfectly
60
Q

An increase in which of the following tends to increase pulse pressure?

A) Systolic pressure
B) Capillary hydrostatic pressure
C) Arterial compliance
D) Stroke volume
E) A and D

A

E) A and D

How well did you know this?
1
Not at all
2
3
4
5
Perfectly
61
Q

Which set of physiological changes would be expected to occur in a person who stands up from a supine position?

A

B) increase, increase, decrease

How well did you know this?
1
Not at all
2
3
4
5
Perfectly
62
Q

Which of the following would have the slowest rate of net movement across the capillary wall?

A) Sodium
B) Albumin
C) Glucose
D) Oxygen

A

B) Albumin

How well did you know this?
1
Not at all
2
3
4
5
Perfectly
63
Q

Which one of the following compensatory physiological changes would be expected to occur in a person who stands up from a supine position?

A) Increased parasympathetic nerve activity
B) Increased sympathetic nerve activity
C) Decreased heart rate
D) Decreased heart contractiltiy

A

B) Increased sympathetic nerve activity

How well did you know this?
1
Not at all
2
3
4
5
Perfectly
64
Q

An increase in which of the following tends to increase capillary filtration rate?

A) Capillary wall hydraulic conductivity
B) Arteriolar resistance
C) Plasma colloid osmotic pressure
D) Interstitial hydrostatic pressure
E) Plasma sodium concentration

A

A) Capillary wall hydraulic conductivity

How well did you know this?
1
Not at all
2
3
4
5
Perfectly
65
Q

The tendency for turbulent flow is greatest in which of the following?

A) Arterioles
B) Capillaries
C) Small arterioles
D) Aorta

A

D) Aorta

How well did you know this?
1
Not at all
2
3
4
5
Perfectly
66
Q

Which pressure is normally negative in a muscle capillary bed in the lower extremities?

A) Plasma colloid osmotic pressure
B) Capillary hydrostatic pressure
C) Interstitial hydrostatic pressure
D) Interstitial colloid osmotic pressure
E) Venous hydrostatic pressure

A

C) Interstitial hydrostatic pressure

How well did you know this?
1
Not at all
2
3
4
5
Perfectly
67
Q

A 60-year-old man has a mean arterial blood pressure of 130 mm Hg, a heart rate of 78 beats/min, a right atrial pressure of 0 mm Hg, and a cardiac output of 3.5 L/min. He also has a pulse pressure of 35 mm Hg and a hematocrit of 40. What is the approximate total peripheral vascular resistance in this man?

A) 17 mm Hg/l/min
B) 1.3 mm Hg/l/ming
C) 13 mm Hg/l/min
D) 27 mm Hg/l/min
E) 37 mm Hg/l/min

A

E) 37 mm Hg/l/min

How well did you know this?
1
Not at all
2
3
4
5
Perfectly
68
Q

What would increase venous hydrostatic pressure in the legs?

A) Decrease in right atrial pressure
B) Pregnancy
C) Decreased movement of leg muscles
D) Abdominal compression of vena cava by a solid tumor in the abdomen
E) B and D

A

E) B and D

How well did you know this?
1
Not at all
2
3
4
5
Perfectly
69
Q

Movement of solutes such as Na+ across the capillary walls occurs primarily by which process?
A) Filtration
B) Active transport
C) Vesicular transport
D) Diffusion

A

D) Diffusion

70
Q

A nitric oxide donor is infused into the brachial artery of a 22- year-old man. Which set of microcirculatory changes would be expected in the infused arm?

A

A) Increase, increase, increase

71
Q

Which pressure is normally negative in a muscle capillary bed in the lower extremities?

A) Plasma colloid osmotic pressure
B) Capillary hydrostatic pressure
C) Interstitial hydrostatic pressure
D) Interstitial colloid osmotic pressure
E) Venous hydrostatic pressure

A

C) Interstitial hydrostatic pressure

72
Q

Which condition often occurs in progressive hemorrhagic shock?

A) Vasomotor center failure
B) Increased urine output
C) Tissue alkalosis
D) Decreased capillary permeability
E) Increased mean systemic filling pressure

A

A) Vasomotor center failure

73
Q

A 50-year-old woman received an overdose of furosemide, and her arterial pressure decreased to 70/40. Her heart rate is 120, and her respiratory rate is 30/min. What therapy would you recommend?

A) Whole blood infusion
B) Plasma infusion
C) Infusion of a balanced electrolyte solution
D) Infusion of a sympathomimetic drug
E) Administration of a glucocorticoid

A

C) Infusion of a balanced electrolyte solution

74
Q

In the above figure, for the cardiac output and venous return curves defined by the solid red lines (with the equilibrium at A), which of the following options is true?

A) Mean systemic filling pressure is 12 mm Hg
B) Right atrial pressure is 2 mm Hg
C) Resistance to venous return is 1.4 mm Hg/l/min
D) Pulmonary arterial flow is approximately 7 l/min
E) Resistance to venous return is 0.71 mm Hg/l/min

A

C) Resistance to venous return is 1.4 mm Hg/l/min

75
Q

A 30-year-old woman comes to a local emergency department with severe vomiting. She has pale skin, tachycardia, an arterial pressure of 70/45, and trouble walking. What therapy do you recommend to prevent shock?

A) Infusion of packed red blood cells
B) Administration of an antihistamine
C) Infusion of a balanced electrolyte solution
D) Infusion of a sympathomimetic drug
E) Administration of a glucocorticoid

A

C) Infusion of a balanced electrolyte solution

76
Q

A 30-year-old man is resting, and his sympathetic output increases to maximal values. Which set of changes would be expected in response to this increased sympathetic output?

A

A) increase, increase, increase

77
Q

If a patient has an oxygen consumption of 240 ml/min, a pulmonary vein oxygen concentration of 180 ml/l of blood, and a pulmonary artery oxygen concentration of 160 ml/l of blood units, what is the cardiac output in l/min?

A) 8
B) 10
C) 12
D) 16
E) 20

A

C) 12

78
Q

What normally causes the cardiac output curve to shift to the right along the right atrial pressure axis?

A) Changing intrapleural pressure to −1 mm Hg
B) Increasing mean systemic filling pressure
C) Taking a patient off a mechanical ventilator and allowingnormal respiration
D) Decreasing intrapleural pressure to −7 mm Hg
E) Breathing against a negative pressure

A

A) Changing intrapleural pressure to −1 mm Hg

79
Q

What normally causes the cardiac output curve to shift to the left along the right atrial pressure axis?

A) Surgically opening the chest
B) Severe cardiac tamponade
C) Breathing against a negative pressure
D) Playing a trumpet
E) Positive-pressure breathing

A

C) Breathing against a negative pressure

80
Q

What will elevate the plateau of the cardiac output curve?

A) Surgically opening the thoracic cage
B) Connecting a patient to a mechanical ventilator
C) Cardiac tamponade
D) Increasing parasympathetic stimulation of the heart
E) Increasing sympathetic stimulation of the heart

A

E) Severe aortic regurgitation

81
Q

What is normally associated with an increased cardiac output?

A) Increased parasympathetic stimulation
B) Atrioventricular (A-V) fistula
C) Decreased blood volume
D) Polycythemia
E) Severe aortic regurgitation

A

B) Atrioventricular (A-V) fistula

82
Q

Which condition would be expected to decrease mean systemic filling pressure?

A) Norepinephrine administration
B) Increased blood volume
C) Increased sympathetic stimulation
D) Increased venous compliance
E) Skeletal muscle contraction

A

D) Increased venous compliance

83
Q

A 35-year-old man undergoes several cardiac test during exercise. The following measurements are made:

What is the resistance to venous return (mm Hg/l/min) in this individual?

A) 0.1
B) 0 .5
C) 1.0
D) 1.4
E) 2.0

A

D) 1.4

84
Q

What is the resistance to venous return (mm Hg/l/min) in this individual?

A) 0.1
B) 0 .5
C) 1.0
D) 1.4
E) 2.0

A

C) 1.0

85
Q

In which condition would you expect a decreased resistance to venous return?

A) Anemia
B) Increased venous resistance
C) Increased arteriolar resistance
D) Increased sympathetic output
E) Obstruction of veins

A

A) Anemia

86
Q

Which of the following would decrease cardiac output?

A) Increased stroke volume
B) Increased heart rate
C) Increased mean systemic filling pressure
D) Increased resistance to venous return
E) Increased venous return

A

D) Increased resistance to venous return

87
Q

In which condition would you normally expect to find a decreased cardiac output?

A) Hyperthyroidism
B) Beriberi
C) A-V fistula
D) Increased muscle mass
E) Hypothyroidism

A

E) Hypothyroidism

88
Q

Which of the following sets of changes would tend to increase coronary blood flow?

A

G) decrease, increase, increase, increase, increase

89
Q

What will usually increase the plateau level of the cardiac output curve?

A) Myocarditis
B) Severe cardiac tamponade
C) Decreased parasympathetic stimulation of the heart
D) Myocardial infarction
E) Mitral stenosis

A

C) Decreased parasympathetic stimulation of the heart

90
Q

If a person has been exercising for 1 hour, which organ will have the smallest decrease in blood flow?

A) Brain
B) Intestines
C) Kidneys
D) Nonexercising skeletal muscle
E) Pancreas

A

A) Brain

91
Q

A 35-year-old man has been diagnosed with a vitamin B1 deficiency. Oxygen consumption in this man is 400 ml/min. In addition, pulmonary vein oxygen concentration is 200 ml/l of blood, and pulmonary artery oxygen concentration is 150 ml/l of blood. What is the cardiac output (l/min) in this man?

A) 4.0
B) 5.0
C) 6.0
D) 7.0
E) 8.0

A

E) 8.0

92
Q

Which vasoactive agent is usually the most important controller of coronary blood flow?

A) Adenosine
B) Bradykinin
C) Prostaglandins
D) Carbon dioxide
E) Potassium ions

A

A) Adenosine

93
Q

What will elevate the plateau of the cardiac output curve?

A) Surgically opening the thoracic cage
B) Connecting a patient to a mechanical ventilator
C) Cardiac tamponade
D) Increasing parasympathetic stimulation of the heart
E) Increasing sympathetic stimulation of the heart

A

E) Increasing sympathetic stimulation of the heart

94
Q

The most likely cause of cardiac pain in acute ischemic coronary
disease is an increase in the extracellular concentration of the following:

A) Adenosine
B) Potassium
C) Nitric oxide
D) ATP
E) Lactic acid

A

E) Lactic acid

95
Q

Which condition normally causes arteriolar vasodilation during
exercise?
A) Decreased plasma potassium ion concentration
B) Increased histamine release
C) Decreased plasma nitric oxide concentration
D) Increased plasma adenosine concentration
E) Decreased plasma osmolality

A

D) Increased plasma adenosine concentration

96
Q

At the onset of exercise, the mass sympathetic nervous system strongly discharges. What would you expect to occur?

A) Increased sympathetic impulses to the heart
B) Decreased coronary blood flow
C) Decreased cerebral blood flow
D) Reverse stress relaxation
E) Venous dilation

A

A) Increased sympathetic impulses to the heart

97
Q

A sudden occlusion that occurs in larger coronary arteries causes an increase in the following:

A) Dilation of small anastomoses in cardiac tissue
B) Increase collateral blood flow
C) Increase production of adenosine
D) All of the above
E) Only A and C

A

D) All of the above

98
Q

A 70-year-old man with a weight of 100 kg (220 lb) and a blood pressure of 160/90 mm Hg has been told by his doctor that he has angina caused by myocardial ischemia. Which treatment would be beneficial to this man?

A) Increased dietary calcium
B) Isometric exercise
C) A beta-1 receptor stimulator
D) Angiotensin II infusion
E) Nitroglycerin

A

E) Nitroglycerin

99
Q

Which of the following is (are) responsible for the increase in stroke volume in response to increased venous return?

A) Stretch of right atrium initiates a nervous reflex called the Bainbridge reflex
B) Stretch of the sinus node in the wall of the right atrium has a direct effect on the rhythmicity of the node to increase the heart rate
C) Frank-Starling law of the heart
D) All of the above
E) A and C

A

D) All of the above

100
Q

Which event normally occurs during exercise?

A) Arteriolar dilation in exercising muscle
B) Decreased sympathetic output
C) Venoconstriction
D) Decreased release of norepinephrine by the adrenals
E) A and C

A

E) A and C

101
Q

Which of the following is (are) responsible for the increase in stroke volume in response to increased venous return?

A) Stretch of right atrium initiates a nervous reflex called the Bainbridge reflex
B) Stretch of the sinus node in the wall of the right atrium has a direct effect on the rhythmicity of the node to increase the heart rate
C) Frank-Starling law of the heart
D) All of the above
E) A and C

A

D) All of the above

102
Q

. A 60-year-old man sustained an ischemia-induced myocardial infarction and died from ventricular fibrillation. In this patient, what factor was most likely to increase the tendency of the heart to fibrillate after the infarction?
A) Low potassium concentration in the heart extracellular fluid
B) A decrease in ventricular diameter
C) Increased sympathetic stimulation of the heart
D) Low adenosine concentration
E) Decreased parasympathetic stimulation of the heart

A

C) Increased sympathetic stimulation of the heart

103
Q

A 60-year-old man has been told by his doctor that he has angina caused by myocardial ischemia. Which treatment would be beneficial to this man?

A) Angiotensin-converting enzyme inhibition
B) Isometric exercise
C) Chelation therapy such as ethylenediamine tetraacetic acid (EDTA)
D) Beta receptor stimulation
E) Increased dietary calcium

A

A) Angiotensin-converting enzyme inhibition

104
Q

What is one of the major causes of death after myocardial infarction?

A) Increased cardiac output
B) A decrease in pulmonary interstitial volume
C) Fibrillation of the heart
D) Increased cardiac contractility

A

C) Fibrillation of the heart

105
Q

Which statement about the results of sympathetic stimulation is most accurate?

A) Epicardial flow increases
B) Venous resistance decreases
C) Arteriolar resistance decreases
D) Heart rate decreases
E) Venous reservoirs constrict

A

E) Venous reservoirs constrict

106
Q

What is normally associated with the chronic stages of compensated heart failure? Assume the patient is resting.

A) Dyspnea
B) Decreased right atrial pressure
C) Decreased heart rate
D) Sweating
E) Increased mean systemic filling pressure

A

B) Decreased right atrial pressure

107
Q

What normally occurs in a person with unilateral left heart failure?

A) Decreased pulmonary artery pressure
B) Decreased left atrial pressure
C) Decreased right atrial pressure
D) Edema of feet
E) Increased mean pulmonary filling pressure

A

E) Increased mean pulmonary filling pressure

108
Q

What normally occurs in a person with unilateral left heart failure?

A) Decreased pulmonary artery pressure
B) Decreased left atrial pressure
C) Decreased right atrial pressure
D) Edema of feet
E) Increased mean pulmonary filling pressure

A

E) Increased mean pulmonary filling pressure

109
Q

What normally causes renal sodium retention during
compensated heart failure?

A) Increased formation of angiotensin II
B) Increased release of atrial natriuretic factor
C) Sympathetic vasodilation of the afferent arterioles
D) Increased glomerular filtration rate
E) Increased formation of antidiuretic hormone (ADH)

A

A) Increased formation of angiotensin II

110
Q

Which condition normally accompanies acute unilateral right
heart failure?

A) Increased right atrial pressure
B) Increased left atrial pressure
C) Increased urinary output
D) Increased cardiac output
E) Increased arterial pressure

A

A) Increased right atrial pressure

111
Q

Which intervention would normally be beneficial to a patient with acute pulmonary edema?

A) Infuse a vasoconstrictor drug
B) Infuse a balanced electrolyte solution
C) Administer furosemide
D) Administer a bronchoconstrictor
E) Infuse whole blood

A

C) Administer furosemide

112
Q

A 60-year-old man had a heart a ack 2 days ago, and his blood pressure has continued to decrease. He is now in cardiogenic shock. Which therapy would be most beneficial?

A) Placing tourniquets on all four limbs
B) Administering a sympathetic inhibitor
C) Administering furosemide
D) Administering a blood volume expander
E) Increasing dietary sodium intake

A

D) Administering a blood volume expander

113
Q

If a 21-year-old male patient has a cardiac reserve of 300% and a maximum cardiac output of 16 l/min, what is his resting cardiac output?

A) 3 l/min
B) 4 l/min
C) 5.33 l/min
D) 6 l/min
E) 8 l/min

A

B) 4 l/min

114
Q

Which of the following occurs during heart failure and causes an increase in renal sodium excretion?

A) Increased aldosterone release
B) Increased atrial natriuretic factor release
C) Decreased glomerular filtration rate
D) Increased angiotensin II release
E) Decreased mean arterial pressure

A

B) Increased atrial natriuretic factor release

115
Q

Which intervention would be appropriate therapy for a patient in cardiogenic shock?

A) Placing tourniquets on the four limbs
B) Withdrawing a moderate amount of blood from the patient
C) Administering furosemide
D) Infusing a vasoconstrictor drug

A

D) Infusing a vasoconstrictor drug

116
Q

Which condition is normally associated with an increase in mean systemic filling pressure?

A) Decreased blood volume
B) Congestive heart failure
C) Sympathetic inhibition
D) Venous dilation

A

A) Decreased blood volume

117
Q

Patients with pulmonary edema often have dyspnea because of
accumulation of fluid in the lungs. Which of the following would normally be the most beneficial for a patient with acute pulmonary edema?

A) Infusing furosemide
B) Infusing dobutamine
C) Infusing saline solution D) Infusing norepinephrine E) Infusing whole blood

A

A) Infusing furosemide

118
Q

Which of the following is associated with compensated heart failure?

A) Increased cardiac output
B) Increased blood volume
C) Decreased mean systemic filling pressure
D) Normal right atrial pressure

A

B) Increased blood volume

119
Q

Which condition normally occurs during the early stages of compensated heart failure?

A) Increased right atrial pressure
B) Normal heart rate
C) Decreased angiotensin II release
D) Decreased aldosterone release
E) Increased urinary output of sodium and water

A

A) Increased right atrial pressure

120
Q

What often occurs during decompensated heart failure?

A) Hypertension
B) Increased mean pulmonary filling pressure
C) Decreased pulmonary capillary pressure
D) Increased cardiac output
E) Increased norepinephrine in the endings of the cardiac
sympathetic nerves

A

B) Increased mean pulmonary filling pressure

121
Q

The fourth heart sound is associated with which mechanism?

A) In-rushing of blood into the ventricles from atrial contraction
B) Closing of the A-V valves
C) Closing of the pulmonary valve
D) Opening of the A-V valves
E) In-rushing of blood into the ventricles in the early to middle part of diastole

A

A) In-rushing of blood into the ventricles from atrial contraction

122
Q

A 40-year-old woman has been diagnosed with a heart murmur. A “ blowing” murmur of relatively high pitch is heard maximally over the left ventricle. The chest radiograph shows an enlarged heart. Arterial pressure in the aorta is 140/40 mm Hg. What is the diagnosis?

A) Aortic valve stenosis
B) Aortic valve regurgitation
C) Pulmonary valve stenosis
D) Mitral valve stenosis
E) Tricuspid valve regurgitation

A

B) Aortic valve regurgitation

123
Q

In which disorder will left ventricular hypertrophy normally occur?

A) Pulmonary valve regurgitation
B) Tricuspid regurgitation
C) Mitral stenosis
D) Tricuspid stenosis
E) Aortic stenosis

A

E) Aortic stenosis

124
Q

An increase in left atrial pressure is most likely to occur in which heart murmur?

A) Tricuspid stenosis
B) Pulmonary valve regurgitation
C) Aortic stenosis
D) Tricuspid regurgitation
E) Pulmonary valve stenosis

A

C) Aortic stenosis

125
Q

A 50-year-old woman at a local hospital has been diagnosed with a heart murmur. A murmur of relatively low pitch is heard maximally over the second intercostal space to the right of the sternum. The chest radiograph shows an enlarged heart. The mean QRS axis of the ECG is −45 degrees. What is the diagnosis?

A) Mitral valve stenosis
B) Aortic valve stenosis
C) Pulmonary valve stenosis
D) Tricuspid valve stenosis
E) Tricuspid valve regurgitation

A

B) Aortic valve stenosis

126
Q

A 40-year-old woman has been diagnosed with a heart murmur of relatively high pitch heard maximally in the second intercostal space to the left of the sternum. The mean QRS axis of his ECG is 150 degrees and the chest radiographs show an enlarged heart. The arterial blood oxygen content is normal. What is the likely diagnosis?

A) Aortic stenosis
B) Aortic regurgitation
C) Pulmonary valve regurgitation
D) Mitral stenosis
E) Tricuspid stenosis

A

C) Pulmonary valve regurgitation

127
Q

Which heart murmur is only heard during diastole?

A) Patent ductus arteriosus
B) Aortic stenosis
C) Tricuspid valve regurgitation
D) Interventricular septal defect
E) Mitral stenosis

A

E) Mitral stenosis

128
Q

Which of the following is associated with the first heart sound?

A) Inrushing of blood into the ventricles as a result of atrial contraction
B) Closing of the A-V valves
C) Closing of the pulmonary valve
D) Opening of the A-V valves
E) Inrushing of blood into the ventricles in the early to middle part of diastole

A

B) Closing of the A-V valves

129
Q
  1. Which heart murmur is only heard during diastole?

A) Patent ductus arteriosus
B) Mitral regurgitation
C) Tricuspid valve stenosis
D) Interventricular septal defect
E) Aortic stenosis

A

C) Tricuspid valve stenosis

130
Q

Which condition often occurs in a person with progressive
hemorrhagic shock?

A) Increased capillary permeability
B) Stress relaxation of veins
C) Tissue alkalosis
D) Increased urine output
E) Increased mean systemic filling pressure

A

A) Increased capillary permeability

131
Q

In which condition will administration of a sympathomimetic drug be the therapy of choice to prevent shock?

A) Spinal cord injury
B) Shock due to excessive vomiting
C) Hemorrhagic shock
D) Shock caused by excess diuretics

A

A) Spinal cord injury

132
Q

Which mechanism is associated with the third heart sound?

A) Inrushing of blood into the ventricles as a result of atrial contraction
B) Closing of the A-V valves
C) Closing of the pulmonary valve
D) Opening of the A-V valves
E) Inrushing of blood into the ventricles in the early to middlepart of diastole

A

E) Inrushing of blood into the ventricles in the early to middle
part of diastole

133
Q

The blood pressure of a 60-year-old man decreased to 55/35 mm Hg during induction of anesthesia. His ECG still shows a normal sinus rhythm. What initial therapy do you recommend?

A) Infusion of packed red blood cells
B) Infusion of plasma
C) Infusion of a balanced electrolyte solution
D) Infusion of a sympathomimetic drug
E) Administration of a glucocorticoid

A

D) Infusion of a sympathomimetic drug

134
Q

A 65-year-old man enters a local emergency department a few minutes after receiving an influenza inoculation. He has pallor, tachycardia, arterial pressure of 80/50, and trouble walking. What therapy do you recommend to prevent shock?

A) Infusion of blood
B) Administration of an antihistamine
C) Infusion of a balanced electrolyte solution such as saline
D) Infusion of a sympathomimetic drug
E) Administration of tissue plasminogen activator

A

D) Infusion of a sympathomimetic drug

135
Q

Which condition often occurs in compensated hemorrhagic shock? Assume systolic pressure is 48 mm Hg.

A) Decreased heart rate
B) Stress relaxation of veins
C) Decreased ADH release
D) Decreased absorption of interstitial fluid through the
capillaries
E) Central nervous system (CNS) ischemic response

A

E) Central nervous system (CNS) ischemic response

136
Q

If a patient undergoing spinal anesthesia experiences a large decrease in arterial pressure and goes into shock, what would be the therapy of choice?

A) Plasma infusion
B) Blood infusion
C) Saline solution infusion
D) Glucocorticoid infusion
E) Infusion of a sympathomimetic drug

A

E) Infusion of a sympathomimetic drug

137
Q

A 25-year-old man who has been in a motorcycle wreck enters the emergency department. His clothes are very bloody, and his arterial pressure is decreased to 70/40 mm Hg. His heart rate is 120 beats/min, and his respiratory rate is 30/min. Which therapy would the physician recommend?

A) Infusion of blood
B) Infusion of plasma
C) Infusion of a balanced electrolyte solution
D) Infusion of a sympathomimetic drug
E) Administration of a glucocorticoid

A

A) Infusion of blood

138
Q

In which type of shock does cardiac output often increase?

A) Hemorrhagic shock
B) Anaphylactic shock
C) Septic shock
D) Neurogenic shock

A

C) Septic shock

139
Q

A 20-year-old man who has been hemorrhaging as a result of a gunshot wound enters a local emergency department. He has pale skin, tachycardia, an arterial pressure of 60/40 mm Hg, and trouble walking. Unfortunately, the blood bank is out of whole blood. Which therapy would the physician recommend to prevent shock?

A) Administration of a glucocorticoid
B) Administration of an antihistamine
C) Infusion of a balanced electrolyte solution
D) Infusion of a sympathomimetic drug
E) Infusion of plasma

A

E) Infusion of plasma

140
Q

What often occurs during progressive shock?

A) Patchy areas of necrosis in the liver
B) Decreased tendency for blood to clot
C) Increased glucose metabolism
D) Decreased release of hydrolases by lysosomes
E) Decreased capillary permeability

A

A) Patchy areas of necrosis in the liver

141
Q

Release of which substance causes vasodilation and increased capillary permeability during anaphylactic shock?

A) Histamine
B) Bradykinin
C) Nitric oxide
D) Atrial natriuretic factor
E) Adenosine

A

A) Histamine

142
Q

A 36-year-old female has a resting cardiac output (CO) of 4.8 l/min and after maximum exercise increased to 19.2 l/min. What is (approximately) her cardiac reserve?

A) 400%
B) 300%
C) 500%
D) Cannot be estimated without mean arterial pressure values
E) Cannot be estimated without total peripheral resistance
values

A

B) 300%

143
Q

A 58-year-old patient with a history of atherosclerosis and hypertension suffers a heart a ttack. What are acute events that take place immediately (0–30 seconds) after heart damage?

A) Increased cardiac output
B) Blood accumulation in the aorta
C) Sympathetic activation
D) Parasympathetic activation
E) Inhibition of angiotensin II

A

C) Sympathetic activation

144
Q

An important reason why moderate fluid retention in low-output heart failure is beneficial is:

A) Preserves isovolumetric contraction
B) Increases afterload
C) Improves preload
D) Reduces aortic pressure
E) Decreases peripheral edema

A

C) Improves preload

145
Q

In decompensated heart failure, the failure of CO to rise enough will
result in:

A) Progressive fluid retention, increased mean filling pressure,
and increased right atrial pressure
B) Progressive parasympathetic activation, decreased
aldosterone, increased heart rate.
C) Moderate fluid retention, increased mean filling pressure,
decreased venous return
D) Vasoconstriction, bronchospasm, and decreased right atrial
pressure

A

A) Progressive fluid retention, increased mean filling pressure,
and increased right atrial pressure

146
Q

A 67-year-old man has an ejection fraction of 0.32, no cyanosis, a history of dilated cardiomyopathy and heart failure, and a systolic murmur. What is your most likely diagnosis?

A) Mitral stenosis
B) Tetralogy of Fallot
C) Mitral regurgitation
D) Patent ductus arteriosus
E) Tricuspid stenosis

A

C) Mitral regurgitation

147
Q

The murmur in mitral stenosis is due to:

A) Increased pulmonary pressures
B) Narrowed outflow tract of the left ventricle
C) Backflow from atria to the pulmonary vessels
D) Narrowed mitral valve opening
E) A and D

A

D) Narrowed mitral valve opening

148
Q

Which of the following is correct about hemorrhagic shock?

A) Deterioration of the heart is probably the most important
factor in progression of shock
B) Deterioration of the liver is probably the most important factor in progression of shock
C) Autoregulation in the brain reverses cellular deterioration in irreversible shock
D) Autoregulation in the heart reverses cellular deterioration in irreversible shock
E) Autoregulation in the kidneys reverses cellular deterioration in irreversible shock

A

A) Deterioration of the heart is probably the most important
factor in progression of shock

149
Q

A 48-year-old male suffers a massive heart a ack that deteriorates over 70% of his left ventricle (LV) (EKG shows ST elevation from V1 to V6, in lead I, and in aVL). The blood pressure is 82/57 mm Hg, heart rate is 135 beats/min, pulse is weak and the patient displays generalized signs of hypoperfusion (lethargic, pale, sweat, cold skin). The diagnosis of the heart a ack was done at his home 7 hours ago. The patient was admi ed 55 minutes ago and has been receiving IV fluids, oxygen, and sympathomimetics He showed a brief improvement in blood pressure and cardiac dynamics but then continued to deteriorate with no further response to treatment. Which of the following statements is most likely to be correct?

A) The delay between diagnosis and admi ance at hospital complicated the patients’ hemorrhagic shock with a progressive decrease in capillary permeability
B) It is possible that fluids may have been insufficient and increased administration of blood + fluids may reverse cellular deterioration
C) The patient is most likely at the irreversible stage of shock
D) The diminished delivery of oxygen to the tissues leads to
generalized tissue alkalosis and cellular deterioration
E) Since the cause of shock is from cardiac origin, no toxins are
released or accumulated and cellular deterioration may not
develop.

A

C) The patient is most likely at the irreversible stage of shock

150
Q

The patient from the previous question, after 2 hours of therapeutic interventions, develops ventricular fibrillation.

Which of the following will represent the most significant change in his left ventricular cardiovascular dynamics?

A) A significant increase in preload
B) A decrease in afterload with preserve preload
C) A circulatory arrest
D) An increased end-systolic volume
E) A shift of the pressure-volume relationship to the left

A

C) A circulatory arrest

151
Q

Which of the following is correct about treatment of shock?

A) Dextran solution (if plasma is not available) is helpful as plasma substitution in cardiogenic shock since it increases interstitial pressure.
B) Anti-histamine agents in anaphylactic shock help to prevent development of disseminated intra-vascular coagulation
C) Dextran solution (if whole blood or plasma are not available) is helpful as plasma substitution since it increases intra- vascular hydrostatic pressure.
D) Dextran solution (if whole blood or plasma are not available) is helpful as plasma substitution since it increases intra- vascular colloid osmotic pressure.
E) Glucocorticoids stimulate release of enzymes from lysosomes that are crucial for tissue protection and to prevent cellular deterioration

A

D) Dextran solution (if whole blood or plasma are not available) is helpful as plasma substitution since it increases intra- vascular colloid osmotic pressure.

152
Q

Which of the following would most likely cause a decrease in the
release of thyroid-stimulating hormone?

A) Decreased iodinase enzyme
B) Decreased iodine pump activity in thyroid gland
C) Decreased body temperature
D) Increased thyrotropin releasing hormone
E) Increased plasma thyroxine by venous infusion

A

E) Increased plasma thyroxine by venous infusion

153
Q

The increased cardiac output caused by elevated circulating levels of thyroid hormones is most likely caused by:

A) Direct actions of thyroid-stimulating hormone on the heart
B) Direct actions of thyroid-stimulating hormone on the brain
C) An increase in the metabolic demand of the tissues
D) An increase in plasma cholesterol and triglycerides
E) An increase in total body weight

A

C) An increase in the metabolic demand of the tissues

154
Q

Which of the following depicts the most likely sequence of events in an individual exposed to cold?

A) ↑Thyrotropin-releasing hormone, ↑thyroid-stimulating hormone, ↑thyroxine
B) ↑Thyrotropin-releasing hormone, ↓thyroid-stimulating hormone, ↑thyroxine
C) ↑Thyroid-stimulating hormone, ↑thyrotropin-releasing hormone, ↑thyroxine
D) ↑Thyroid-stimulating hormone, ↓thyrotropin-releasing hormone, ↑thyroxine
E) ↑Thyroxine, ↑thyrotropin-releasing hormone, ↑thyroid- stimulating hormone

A

A) ↑Thyrotropin-releasing hormone, ↑thyroid-stimulating hormone, ↑thyroxine

155
Q

In an individual with a thyroid hormone producing adenoma, one might expect which of the following?

A) ↑ T4, ↓ T3, ↓ TRH, ↓ TSH
B) ↑ T4, ↑ T3, ↓ TRH, ↓ TSH
C) ↑ T4, ↑ T3, ↑ TRH, ↓ TSH
D) ↑ T4, ↑ T3, ↓ TRH, ↑ TSH
E) ↓ T4, ↑ T3, ↓ TRH, ↓ TSH

A

B) ↑ T4, ↑ T3, ↓ TRH, ↓ TSH

156
Q

You suspect thyroid disease in a female patient. Based on the plasma values below, which of the following would be expected?

A) Graves’ disease
B) Secondary hyperthyroidism
C) Hashimoto’s disease
D) Secondary hypothyroidism
E) Euthyroid pregnant

A

C) Hashimoto’s disease

157
Q

A 25-year-old man is severely injured when hit by a speeding vehicle and loses 20% of his blood volume. Which set of physiological changes would be expected to occur in response to the hemorrhage?

A

A) decrease, decrease, increase

158
Q

Which finding is most likely in a patient who has myxedema?

A) Somnolence
B) Palpitations
C) Increased respiratory rate
D) Increased cardiac output
E) Weight loss

A

A) Somnolence

159
Q

A 37-year-old woman presents to her physician with an enlarged thyroid gland and high plasma levels of T4 and T3. Which of the following is likely to be decreased?

A) Heart rate
B) Cardiac output
C) Peripheral vascular resistance
D) Ventilation rate
E) Metabolic rate

A

C) Peripheral vascular resistance

160
Q

Which physiological response is greater for T3 than for T4?

A) Secretion rate from the thyroid
B) Plasma concentration
C) Plasma half-life
D) Affinity for nuclear receptors in target tissues
E) Latent period for the onset of action in target tissues

A

D) Affinity for nuclear receptors in target tissues

161
Q

A number of normal physiological changes occur during pregnancy. Which of the following best describes one of these changes in the mother?

A) Increase total peripheral resistance
B) Increased cardiac output
C) Decreased metabolic rate
D) Decreased body weight
E) Decreased uterine size

A

B) Increased cardiac output

162
Q

A patient is administered sufficient T4 to increase plasma levels of the hormone several fold.
Which set of changes is most likely in this patient after several weeks of T4 administration?

A

B) increase, increase, decrease

163
Q

A 46-year-old man has “puffy” skin and is lethargic. His plasma TSH concentration is low and increases markedly when he is given TRH. What is the most likely diagnosis?

A) Hyperthyroidism due to a thyroid tumor
B) Hyperthyroidism due to an abnormality in the hypothalamus
C) Hypothyroidism due to an abnormality in the thyroid
D) Hypothyroidism due to an abnormality in the hypothalamus
E) Hypothyroidism due to an abnormality in the pituitary

A

D) Hypothyroidism due to an abnormality in the hypothalamus

164
Q

Which of the following would most likely cause a decrease in the release of thyroid-stimulating hormone?

A) Decreased iodinase enzyme
B) Decreased iodine pump activity in thyroid gland
C) Decreased body temperature
D) Increased thyrotropin releasing hormone
E) Increased plasma thyroxine by venous infusion

A

E) Increased plasma thyroxine by venous infusion

165
Q

The increased cardiac output caused by elevated circulating levels of thyroid hormones is most likely caused by:

A) Direct actions of thyroid-stimulating hormone on the heart
B) Direct actions of thyroid-stimulating hormone on the brain
C) An increase in the metabolic demand of the tissues
D) An increase in plasma cholesterol and triglycerides
E) An increase in total body weight

A

C) An increase in the metabolic demand of the tissues

166
Q

A 30-year-old woman reports to the clinic for a routine physical examination. The examination reveals she is pregnant. Her plasma levels of TSH are high, but her total thyroid hormone concentration is normal. Which of the following best reflects the patient’s clinical state?

A) Graves’ disease
B) Hashimoto’s disease
C) A pituitary tumor secreting TSH
D) A hypothalamic tumor secreting thyrotropin-releasing hormone (TRH)

A

B) Hashimoto’s disease

167
Q

A patient has hyperthyroidism due to a pituitary tumor. Which set of physiological changes would be expected?

A

B) Increase, increase, negative

168
Q

A 37-year-old woman presents to her physician with an enlarged thyroid gland and high plasma levels of T4 and T3. Which of the following is likely to be decreased?

A) Heart rate
B) Cardiac output
C) Peripheral vascular resistance
D) Ventilation rate
E) Metabolic rate

A

C) Peripheral vascular resistance

169
Q

Which physiological response is greater for T3 than for T4?

A) Secretion rate from the thyroid
B) Plasma concentration
C) Plasma half-life
D) Affinity for nuclear receptors in target tissues
E) Latent period for the onset of action in target tissues

A

E) Latent period for the onset of action in target tissues

170
Q

An experiment was conducted in which rats were injected with one of two hormones or saline solution (control) for 2 weeks. Autopsies were then performed, and organ weights were measured (in milligrams). Use this information to answer Questions 151 and 152.

What is hormone 1?

A) TRH
B) TSH
C) T4
D) ACTH E) Cortisol

A

B) TSH